math1998

New Member
ارسال ها
336
لایک ها
224
امتیاز
0
پاسخ : ماراتن نظریه ی اعداد (سطح ممتاز)

ممنون که گفتید.الان خودم اینجاش گیج شدم!!!فکر کردم t اوله.چون اگه اول باشه اونو داریم.

همین تیکه آخرشو یه جور دیگه میگم.امیدوارم این دیگه درست باشه.

این رو داریم که
باید مربع کامل باشه.پس
مربع کامله.

حالا اگه داشته باشیم اونوقت چون مربع کامل بعد
,
هه پس باید داشته باشیم:

که اینم چون اشتباهه.چون اگه q مثلا 3 باشه: که اگه طرف چپ رو بزرگش هم کنیم منفی میشه.

حالا اگه داشته باشیم
اونوقت دوباره مثل قبلیه باید داشته باشیم: که اینم اشتباهه.

پس باید داشته باشیم


این یکی چی؟درسته؟
منظورتون اینه که
باید کوچکتر از
باشه؟؟!!
 

m-saghaei

New Member
ارسال ها
338
لایک ها
258
امتیاز
0

math1998

New Member
ارسال ها
336
لایک ها
224
امتیاز
0
پاسخ : ماراتن نظریه ی اعداد (سطح ممتاز)

نه.باید بزرگتر تر از اون باشه.
با حرف شما باید داشته باشیم
درست میگید که میتونیم
رو بزرگ کنیم اما اگه
رو چطور توضیح میدی ؟؟!!!
 

m-saghaei

New Member
ارسال ها
338
لایک ها
258
امتیاز
0
پاسخ : ماراتن نظریه ی اعداد (سطح ممتاز)

با حرف شما باید داشته باشیم
درست میگید که میتونیم
رو بزرگ کنیم اما اگه
رو چطور توضیح میدی ؟؟!!!
نه دیگه.من دارم میگم طرف چپ باید بزرگتر از طرف راست باشه.یعنی
 

math1998

New Member
ارسال ها
336
لایک ها
224
امتیاز
0
پاسخ : ماراتن نظریه ی اعداد (سطح ممتاز)

نه دیگه.من دارم میگم طرف چپ باید بزرگتر از طرف راست باشه.یعنی
ببخشید علامت نامساویو اشتباه گذاشتم اما در کل شما از همین رابطه ای که نوشتید هم نمیتونید نتیجه خوبی بگیرید برای اینکه هیچ اطلاعی از
ندارید!!!
 

m-saghaei

New Member
ارسال ها
338
لایک ها
258
امتیاز
0
پاسخ : ماراتن نظریه ی اعداد (سطح ممتاز)

ببخشید علامت نامساویو اشتباه گذاشتم اما در کل شما از همین رابطه ای که نوشتید هم نمیتونید نتیجه خوبی بگیرید برای اینکه هیچ اطلاعی از
ندارید!!!
یعنی هیچ جوره نمیشه گفت اون نامساویه غلطه؟
 

math1998

New Member
ارسال ها
336
لایک ها
224
امتیاز
0
پاسخ : ماراتن نظریه ی اعداد (سطح ممتاز)

یعنی هیچ جوره نمیشه گفت اون نامساویه غلطه؟
شما باید اطلاع اندکی هم که شده درباره
داشته باشید که اینطور نیست !!!
 

m-saghaei

New Member
ارسال ها
338
لایک ها
258
امتیاز
0
پاسخ : ماراتن نظریه ی اعداد (سطح ممتاز)

شما باید اطلاع اندکی هم که شده درباره
داشته باشید که اینطور نیست !!!
اطلاعات اندک رو داریم اینکه اف تی از تی بزرگتره.
 

حمید آنالیز

Well-Known Member
ارسال ها
1,351
لایک ها
1,322
امتیاز
113
پاسخ : ماراتن نظریه ی اعداد (سطح ممتاز)

سوال رو درست حل کردین سوال بعد : حد اکثر kرا بیابید وقتی که حاصل عبارت روبرو اول بشه با شرط این کهk<2015
k^2+(k-1)^2+(k-2)^2+...+[k/2]^2
 

m-saghaei

New Member
ارسال ها
338
لایک ها
258
امتیاز
0
پاسخ : ماراتن نظریه ی اعداد (سطح ممتاز)

سوال رو درست حل کردین سوال بعد : حد اکثر kرا بیابید وقتی که حاصل عبارت روبرو اول بشه با شرط این کهk<2015
k^2+(k-1)^2+(k-2)^2+...+[k/2]^2
هنوز جواب سوال قبلی تموم نشده.
 

حمید آنالیز

Well-Known Member
ارسال ها
1,351
لایک ها
1,322
امتیاز
113
پاسخ : ماراتن نظریه ی اعداد (سطح ممتاز)

چرا دیگه اون سوالو که به یه جایی رسوندین سوال من حرف نداره ها روش فکر کنین
:8:
 

Dadgarnia

New Member
ارسال ها
1,350
لایک ها
1,127
امتیاز
0
پاسخ : ماراتن نظریه ی اعداد (سطح ممتاز)

سوال رو درست حل کردین سوال بعد : حد اکثر kرا بیابید وقتی که حاصل عبارت روبرو اول بشه با شرط این کهk<2015
k^2+(k-1)^2+(k-2)^2+...+[k/2]^2

k زوج یا فرد باشه در هر دو حالت عبارت بالا تجزیه میشه مگر اینکه k یک یا 2 یا 3 یا 4 یا 5 یا 6 یا 11 یا 12 باشه که به راحتی میشه چک کرد. در ضمن شما قوانین ماراتن رو نخوندید تا سوالی وجود داره نباید سوال جدیدی گذاشته بشه و بهتر بود این سوالو توی بخش نظریه اعداد مقدماتی مطرح می کردین.
 

sepidfekr

New Member
ارسال ها
711
لایک ها
637
امتیاز
0
پاسخ : ماراتن نظریه ی اعداد (سطح ممتاز)

چرا دیگه اون سوالو که به یه جایی رسوندین سوال من حرف نداره ها روش فکر کنین
:8:
خب چرا اینقدر عجله داری هنوز آخر سوال قبلی یکم میلنگه به زودی حل میشه عجله نکن:)
 

m-saghaei

New Member
ارسال ها
338
لایک ها
258
امتیاز
0
پاسخ : ماراتن نظریه ی اعداد (سطح ممتاز)

شما فقط برای اعداد اول اینو ثابت کردین.
نه.اول که گفتم اف پی مساوی پیه اون تموم شد.
بعدش اومدم گفتم دوحالت میگیریم.
1-اف تی بزرگتر از تی باشه.
2-اف تی کوچکتر از تی باشه.
که باید یه جوری این دوتا رو نقض کنیم.
اونی که گفتم داریم اف تی بزرگتر از تیه واسه حالت اوله.
 

AHZolfaghari

Well-Known Member
ارسال ها
935
لایک ها
1,654
امتیاز
93
پاسخ : ماراتن نظریه ی اعداد (سطح ممتاز)

برای سوال قبلی . ابتدا چند تا لم :
الف )
به ازای همه a های طبیعی مربع کامل هستش .
فرض خلف فرض کنید یه p اول وجود داره که عوامل اولش تو
فرد باشه مثلا 2n+1
حالا b رو بدین
پس یه مربع کامل داریم که عوامل p در آن 2n+1 تا است که تناقض پس
مربع کامل به ازای همه a های طبیعی
ب) اگه

فرض خلف
فرض کنید
دارای 2n عامل اول p باشه که همش برای f(x) هست طبیعتا.
حالا b رو بدین
و به تناقض برسید .
ج )

فرض خلف
یه عامل p داشته باشه پس p عاد میکنه یکو که تناقضه پس

د)

اگه
یه عامل اول به نام q داشته باشه q باید عامل p باشه یعنی p=q پس
یه توانی از p است . از اونجایی که
مربع کامله پس

حالا a رو یک بدید و b رو p بدید و عبارت حاصله رو بین دو تا مربع کامل بندازید و نتیجه بگیرید k=0
ه )

حالا نوبت فتح الفتوحه !!
b رو بدید p و a رو بدید n حالا
,

فاصله این دو تا مربع کامل حداقل 2y+1 هست . ( با فرض y<x )
یعنی
اما این مطلب به ازای p بزرگ صادق نیست . حالت x<y هم به همین ترتیب رد میشه پس x=y پس حکم مورد نظر ثابت شد
 

m-saghaei

New Member
ارسال ها
338
لایک ها
258
امتیاز
0
پاسخ : ماراتن نظریه ی اعداد (سطح ممتاز)

b رو بدید p و a رو بدید n حالا
,

فاصله این دو تا مربع کامل حداقل 2y+1 هست . ( با فرض y<x )
یعنی
اما این مطلب به ازای p بزرگ صادق نیست . حالت x<y هم به همین ترتیب رد میشه پس x=y پس حکم مورد نظر ثابت شد
منم منظورم دقیقا همین بود.فقط اینکه p بزرگ یعنی چی؟
 

AHZolfaghari

Well-Known Member
ارسال ها
935
لایک ها
1,654
امتیاز
93
پاسخ : ماراتن نظریه ی اعداد (سطح ممتاز)

منم منظورم دقیقا همین بود.فقط اینکه p بزرگ یعنی چی؟
ببخشید کجای راه حل شما شبیه من بود ؟؟؟ بعدشم کل دغدغه سوال بدست اوردن f اعداد اول هستش . ثانیا سوال الکی نیست که با یکی دو تا کار مبتدیانه حل شه سوال اخر امتحان اخر TST ایران 2011 بوده
 

math1998

New Member
ارسال ها
336
لایک ها
224
امتیاز
0
پاسخ : ماراتن نظریه ی اعداد (سطح ممتاز)

یه نفر سوال بعد رو بذاره!!!
 
بالا